Difference between revisions of "2004 AMC 10B Problems/Problem 5"
(New page: D =0; A must be determined either 3 or 2. 3^2 > 2^3, thus A is 3. B then follows to be 2 and C = 1. The result is 9, (D)) |
(Redirecting to 2004 AMC 12B Problems/Problem 2) |
||
Line 1: | Line 1: | ||
− | + | #redirect [[2004 AMC 12B Problems/Problem 2]] | |
− | |||
− | |||
− | |||
− |
Latest revision as of 12:22, 7 February 2009
Redirect to: